KCET 2022 Mathematics Questions with Answers Key Solutions

Solution:

$A\, K\, M \,S$
$A \rightarrow 3 !$
$K \rightarrow 3 !$
$M \rightarrow 3 !$
$SAKM \rightarrow \frac{1}{19}$

Solution:

$\frac{ a _{3}}{ a _{1}}=25$
$\frac{ ar ^{2}}{ a }=25$
$r ^{2}=5^{2}$
$\frac{ a _{4}}{ a _{5}}=\frac{ ar ^{8}}{ ar ^{4}}= r ^{4}=5^{4}$

Solution:

Given $2 x-3 y+17=0$
is a line perpendicular to line passing through point $P (7,17) \& Q (15, \beta)$ then equation of line passing throught $P \& Q$ is
$y-17=\left(\frac{\beta-17}{15-7}\right)(x-7)$
$8 y-136=\beta x-7 \beta-17 x+119$
$(7-\beta) x+8 y-255=0 \ldots \ldots \ldots . .(2)$
Line (1) \& (2) are perpendicular to each other then
$m _{1} m _{2}=1$
Product of slopes of line (1) \& (2) is $-1$
Now,
$m _{1}=\frac{2}{3}$ and $m _{2}=\frac{\beta-7}{8}$
$m _{1} m _{2}=-1 \Rightarrow \frac{2}{3}\left(\frac{\beta-7}{48}\right)=-1$
$ \beta-7=-12 $
$ \beta=-12+7=-5 $
Hence $[\beta=-5]$.

Solution:

$\alpha=\displaystyle\lim _{x \rightarrow 2^{-}} f(x)=\displaystyle\lim _{x \rightarrow 2} x^{2}-1=3$
$\beta=\displaystyle\lim _{x \rightarrow 2^{+}} f(x)=\displaystyle\lim _{x \rightarrow 2} 2 x+3=7$
$x^{2}-(\alpha+\beta) x+\alpha \beta=0$
$x^{2}-10 x+21=0$

Solution:

$3 x=6$
$x=2 $
$4 x-y=-1$
$8-y=-1$
$9= y$

Solution:

$\sigma^{2}=5, \overline{ x }=\frac{ k }{4}$
$\frac{1}{4}\left(1+0+1+ k ^{2}\right)-\frac{ k ^{2}}{16}=5$
$\frac{ k ^{2}+2}{4} \frac{- k ^{2}}{16}=5$
$\frac{4 k ^{2}+8- k ^{2}}{16}=5 $
$\Rightarrow 3 k ^{2}+8=80$
$3 k ^{2}=72$
$k ^{2}=24$
$k =\pm \sqrt{24}=2 \sqrt{6}$

Solution:

$n ( A ) \neq n ( B )$
Number of bijections is zero

Solution:

$f(-1)=3(-1)=-3$
$f(2)=2^{2}=4$
$f(4)=2(4)=8$
$f(-1)+f(2)+f(4)$
$=-3+4+8=9$

Solution:

$2 b =27-3 a$
$b =\frac{27-3 a }{2}$
$R =\{(1,2),(3,9),(5,6),(7,3)\}$

Solution:

$\displaystyle\lim _{y \rightarrow 0} \frac{3+y^{3}-3}{y\left(\sqrt{3+y^{3}}+\sqrt{3}\right)}=\frac{1}{2 \sqrt{3}}$

Solution:

$\left( A ^{2}\right)^{-1}=\left( A ^{-1}\right)^{2}$

Solution:

$A=\begin{bmatrix}2 & -1 \\ 3 & -2\end{bmatrix}$
$A^{-1}=\frac{1}{-1}\begin{bmatrix}-2 & 1 \\ -3 & 2\end{bmatrix}=\begin{bmatrix}2 & -1 \\ 3 & -2\end{bmatrix}=A$
$A^{2}=\begin{bmatrix}2 & -1 \\ 3 & -2\end{bmatrix}\begin{bmatrix}2 & -1 \\ 3 & -2\end{bmatrix}$
$=\begin{bmatrix}4-3 & -2+2 \\ 6-6 & -3+4\end{bmatrix}=\begin{bmatrix}1 & 0 \\ 0 & 1\end{bmatrix}=I$
$\therefore A^{3}=A$

Solution:

$A ^{ T }=- A$ or $A ^{ n }$ is slow symmetric if $n$ is odd
$P = A ^{2021}$
$P ^{ T }=\left[ A ^{2021}\right]^{ T }=\left[ A ^{ T }\right]^{2021}=(- A )^{2021}=- P$

Solution:

$A =\begin{bmatrix}0 & 1 \\ 0 & 0\end{bmatrix}$
${[ aI + bA ]^{1}=\begin{bmatrix} a & 0 \\ 0 & a \end{bmatrix}+\begin{bmatrix}0 & b \\ 0 & 0\end{bmatrix}=\begin{bmatrix} a & b \\ 0 & a \end{bmatrix} }$
${[ aI + IA ]^{2}=\begin{bmatrix} a & b \\ 0 & a \end{bmatrix}\begin{bmatrix} a & b \\ 0 & a \end{bmatrix}=\begin{bmatrix} a ^{2} & 2 ab \\ 0 & a ^{2}\end{bmatrix} }$
${[ aI + bA ]^{3}=\begin{bmatrix} a ^{2} & 2 ab \\ 0 & a ^{2}\end{bmatrix}\begin{bmatrix}a & b \\ 0 & a \end{bmatrix}=\begin{bmatrix} a ^{3} & 3^{2} b \\ 0 & a ^{3}\end{bmatrix} }$
$\therefore[ aI + bA ]^{ n }=\begin{bmatrix} a ^{ n } & na a ^{ n -1} b \\ 0 & a ^{ n }\end{bmatrix}$
$= a ^{ n } I + n \cdot a ^{ n -1} bA$

Solution:

$A_{3 \times 3}$ matrix $|5 . \operatorname{AdjA}|=5$
$\Rightarrow 5^{3}| A |^{2}=5 $
$\Rightarrow\left| A ^{2}\right|=\frac{1}{5^{2}}$
$| A |=\pm \frac{1}{5}$

Solution:

$\Delta=1\left(2 a^{2}+4\right)+2(4 a-0)+5(8)=86$
$2 a^{2}+8 a+44-86=0$
$2 a^{2}+8 a-42=0$
$a^{2}+4 a-21=0$
Sum of numbers $=-4\left(\therefore-\frac{ b }{ a }=\alpha+\beta\right)$

Solution:

$\Delta=\frac{1}{2}\begin{vmatrix}-2-3 & -2-1 \\ 6+6 & 6-5\end{vmatrix}=\frac{1}{5}\begin{vmatrix}-5 & -3 \\ 12 & 1\end{vmatrix}$
$=\frac{1}{2}|-5+36|=\frac{31}{2}=15.5$

Solution:

$\cos ^{-1}[x]$
$-1 \leq[x] \leq 1 $
$\Rightarrow[x]=\{-1,0,1\}$
$x \in[-1,2)$

Solution:

$y=\left(1+x^{2}\right) \tan ^{-1} x-x$
$\frac{d y}{d x}=\frac{\left(1+x^{2}\right)}{1+x^{2}}+\tan ^{-1} x \cdot(2 x)-1$
$=2 x \tan ^{-1} x$

Solution:

$x = e ^{\theta} \sin \theta=1$
$y = e ^{\theta} \cos \theta=1, \frac{ x }{ y }=\tan \theta=1 $
$\Rightarrow \theta=\frac{\pi}{4}$
$\frac{ dy }{ dx }=\left|\frac{ dy / d \theta}{ dx / d \theta}\right|=\frac{- e ^{\theta} \sin \theta+\cos \theta \cdot e ^{\theta}}{ e ^{\theta} \cos \theta+\sin \theta e ^{\theta}}=\frac{\cos \theta-\sin \theta}{\cos \theta+\sin \theta}$
$=\tan \left(\frac{\pi}{4}-\theta\right)=0$

Solution:

$Y = e^{\sqrt{x\sqrt{x\sqrt{x}}} ........} = e^{x^{\frac{1}{2}} x^{\frac{1}{4}},x^{\frac{1}{8}} ..... = e^{x^{\frac{1}{2}} x^{\frac{1}{4}},x^{\frac{1}{8}} .....}} $
$e ^{ x ^{\frac{1}{2}\left[1+\frac{1}{2}+\frac{1}{4}+\ldots\right]}}= e ^{ x ^{\frac{1}{2}}}= e ^{ x ^{1}}= e ^{ x }$
$\frac{d y}{d x}=e^{x}$
$\frac{ d ^{2} y }{ dx x ^{2}}= e ^{ x } x \log _{ e }^{3}= e ^{\log _{ e }^{3}}=$

Solution:

$f (1)=1, f ^{\prime}(1)=3$
$\frac{ d }{ dx }\left[ f ( f ( f ( x )))+( f ( x ))^{2}\right]$
${\left[ f ^{\prime}( f ( f ( x ))) \cdot f ^{\prime} f ( x ) \cdot f ^{\prime}( x )+2 f ( x ) \cdot f ^{\prime}( x )\right] }$
$= f ^{\prime}( f ( f (1))) f ^{\prime}( f (1)) \cdot f ^{\prime}(1)+2 f (1) \cdot f ^{\prime}(1)$
$f ^{\prime}( f (1)) f ^{\prime}(1) \cdot 3+2 \cdot(1) 3$
$= f ^{\prime}(1) \cdot 3 \cdot 3+6$
$=27+6=33$

Solution:

$y=x^{\sin x}+(\sin x)^{x}$
$\frac{d y}{d x}=\left[x^{\sin x}\right]\left[\frac{\sin x}{x}+\cos x \cdot \log x\right]+$
$(\sin x)^{x}[x \cos x+\log \sin x]$
$x=\frac{\pi}{2}$
$=\frac{\pi}{2}\left[\frac{2}{\pi}\right]+1[0+0]=1$

Solution:

$A_{n}=\begin{bmatrix}1-n & n \\ n & 1-n\end{bmatrix}$
$\left|A_{n}\right|=(1-n)^{2}-n^{2}$
$=1+ n ^{2}-2 n - n ^{2}$

Solution:

$f^{\prime}(x)=\frac{x^{2}}{(x+1)(2+x)^{2}}>0$
$x+1>0 \Rightarrow x>-1$

Solution:

$\frac{d y}{d x}=-\sqrt{\frac{y}{x}}=-1$
$y=x$
$\sqrt{x}+\sqrt{x}=6$
$x=9, y=9$

Solution:

$f^{\prime}(x)=\left(12 \sin ^{2} x-12 \sin x+12\right) \cos x$
$f^{\prime}(x)=12\left(\sin ^{2} x-\sin x+1\right) \cos x$
$\sin ^{2} x-\sin x+1>0$
$x \in\left(\frac{\pi}{2}, \pi\right) \cos x < 0$

Solution:

$\int\limits_{0}^{8}[x] d x=1+2+3+\ldots .+7$
$=\frac{7(7+1)}{2}=28$

Solution:

Put $\sin \theta= t$
$\int\limits_{0}^{1} t^{1 / 2}\left(1-t^{2}\right) d t=\frac{8}{21}$

Solution:

$x=0 \Rightarrow y=1$
$\frac{d y}{d x}=\frac{-y}{e^{y}+x}$
$\left(\frac{d y}{d x}\right)_{(0,1)}=-\frac{1}{e}$
$\left(\frac{d^{2} y}{d x^{2}}\right)_{(0,1)}=\frac{1}{e^{2}}$

Solution:

$2 \int \frac{\cos ^{2} x-\cos ^{2} \alpha}{\cos x-\cos \alpha} d x=2 \int(\cos x+\cos \alpha) d x$
$=2[\sin x+x \cos \alpha]$

Solution:

$\int\limits_{0}^{1} e^{x}\left[\frac{1}{(x+2)^{2}}-\frac{2}{(x+2)^{3}}\right] d x$
$=\left[\frac{e^{x}}{(x+2)^{2}}\right]_{0}^{1}=\frac{e}{9}-\frac{1}{4}$

Solution:

$\frac{1}{(x+2)\left(x^{2}+1\right)}=\frac{A}{x+2}+\frac{B x+C}{x^{2}+1}$
$A=\frac{1}{5}, B=\frac{-1}{5}, C=\frac{2}{5}$

Solution:

$A=\int\limits_{0}^{\pi / 3} \tan x d x=\log |\sec x|_{0}^{\pi / 3}=\log 2$

Solution:

$I=\left[\frac{x^{3}}{3}\right]_{2}^{3}=\frac{1}{3}(27-8)=\frac{19}{3}$

Solution:

$\sin x=t$
$\int\limits_{0}^{\frac{\pi}{2}} \frac{\cos x \sin x}{1+\sin x} d x $
$\Rightarrow \int\limits_{0}^{1} \frac{t}{1+t} d t=1-\log 2$

Solution:

$y \cdot x=\frac{x^{4}}{4}+C$
$2 y(2)-y(1)=\frac{15}{4}$

Solution:

$x+y=z \Rightarrow \frac{d z}{d x}=1+z^{2}$
$\int \frac{1}{1+z^{2}} d z=\int 1 d x$
$\operatorname{Tan}^{-1}(x+y)=x+c$

Solution:

$I . F=\log x$,
$y \log x=2 x(\log x-1)+c$
If $x=e$ then $y=c$ then $y(e)=2 e$

Solution:

$\left(1+ y _{1}^{2}\right)=\left( y _{2}\right)^{3}$
$2+3=5$

Solution:

verification $(2,-3,4)$

Solution:

$\cos \theta=\frac{3 \times 1+5 \times 4+4 \times 2}{\sqrt{3^{2}+5^{2}+4^{2}} \times \sqrt{1^{2}+4^{2}+2^{2}}}=\frac{31}{5 \sqrt{42}}$
$\theta=\cos ^{-1} \frac{31}{5 \sqrt{42}}$

Solution:

At $(0,2),(3,0), z=12$
Hence minimum at $2$ points.

Solution:

Distance $=\frac{|2-2-4-4|}{\sqrt{1+4+16}}=\frac{(8)}{\sqrt{21}}$

Solution:

X 0 1 2 3
$P(x)$ $\frac{1}{8}$ $\frac{3}{8}$ $\frac{3}{8}$ $\frac{1}{8}$

Mean $=\frac{3}{8}+\frac{6}{8}+\frac{3}{8}=\frac{12}{8}=\frac{3}{2}=1.5$

Solution:

$P ( A )=\frac{1}{2}, P ( B )=\frac{1}{3}, P ( A \cap B )=\frac{1}{4} \times \frac{1}{3}=\frac{1}{12}$
$P ( A \cap B )= P (\overline{ A \cup B })$
$P ( A \cap B )=1- P ( A \cup B )$
$P ( A \cap B )=1-\left[\frac{1}{2}+\frac{1}{3}-\frac{1}{12}\right]$
$P ( A \cap B )=1-\left[\frac{6+4-1}{12}\right]$
$P ( A \cap B )=1-\frac{9}{12}$
$P ( A \cap B )=\frac{1}{4}$

Solution:

$P \left( E _{1}\right)=$ probability of there is lockdown $=0.7$
$P\left(E_{2}\right)=$ probability of there is lockdown $=0.3$
$A$ is the event controlled in one month
$P \left( A / E _{1}\right)=0.8, P \left( A / E _{2}\right)=0.3$
$P(A)=0.7(0.8)+(0.3)(0.3) $
$=0.56+0.09=0.65$

Solution:

$P ( A )=\frac{1}{4}, P ( B )=\frac{3}{4}, P ( A \cup B )=\frac{13}{20}$
$\frac{1}{4}+ x -\frac{1}{4} \cdot x =\frac{13}{20}$
$\frac{3}{4} x-\frac{13}{20}-\frac{5}{20}=\frac{8}{20}$
$x=\frac{8}{20} \times \frac{4}{3}=\frac{8}{15}$

Solution:

$n ( A )= p , n ( B )= q$
$n ( A \times B )=7$
$pq =7$
$p ^{2}+ q ^{2}=7^{2}+1^{2}$ or $1^{2}+7^{2}$
$p ^{2}+ q ^{2}=50$

Solution:

$1 - x > 0 , 1 - x \neq 1$
$x -1<0 \,\,\,\,\,\,x \neq 0 \,\,\,\,\,\,x +2 \geq 0$
$x <1 \,\,\,\,\,\,x \geq-2$
$\therefore x \in[-2,0) \cup(0,1)$

Solution:

By graph

Solution:

$\frac{\pi}{32}=\frac{180^{\circ}}{32}=5^{0} 37^{\prime} 30^{\prime \prime}$

Solution:

$\sin \frac{5 \pi}{12} \cdot \sin \frac{\pi}{12}$
$=\frac{1}{2} \sin \frac{\pi}{6}$
$=\frac{1}{2}+\frac{1}{2}=\frac{1}{4}$

Solution:

$1+\cos \theta=2 \cos ^{2}\left(\frac{\theta}{2}\right)$
$\sqrt{2+\sqrt{2+\sqrt{2+2 \cos 8 \theta}}}=2 \cos \theta$

Solution:

Odd number $=\{1,3,5,7,9\}$
No. of sub sets $=2^{5}=32$